Number of compositions of $n$ such that each term is less than equal to $k.$












4












$begingroup$


Let $n$ be an integer $geq 1.$ Then a partition of $n$ is a sequence of positive integers (greater than equal to $1$) such that their sum equals $n.$ So for instance if $n=4$ then
$$[[4], [1, 3], [1, 1, 2], [1, 1, 1, 1], [1, 2, 1], [2, 2], [2, 1, 1], [3, 1]]$$
is a collection of all partitions of $n$ with order. Clearly for any $n$ the number of such ordered partitions is $2^{n-1}.$ However I want to count the number of paritions of $n$ where each integer in the parition is less than equal to some integer $k.$ So for instance if $n=4$ and $k=2$ then
$$[[1, 1, 2], [1, 1, 1, 1], [1, 2, 1], [2, 2], [2, 1, 1]]$$
is a collection of all the $2$-partitions of $4.$ Is there a general formula for finding this or maybe an asymptotic expression?










share|cite|improve this question











$endgroup$












  • $begingroup$
    See this, specifically $A$-restricted compositions.
    $endgroup$
    – MathematicsStudent1122
    Dec 28 '18 at 21:48












  • $begingroup$
    Well, for $k=2$ you get the Fibonacci sequence, so if there is a general formula, it will be complicated for sure.
    $endgroup$
    – SmileyCraft
    Dec 28 '18 at 21:48










  • $begingroup$
    @MathematicsStudent1122 I saw this, but it does not give me the answer to my question. Maybe I am missing something. Perhaps you could elaborate?
    $endgroup$
    – model_checker
    Dec 28 '18 at 21:50










  • $begingroup$
    You can convert them into eqs. and find no. of integral solutions here is a link of finding no. of possible integral solutions math.stackexchange.com/a/2990467/584828
    $endgroup$
    – Mustang
    Dec 28 '18 at 21:52










  • $begingroup$
    @SmileyCraft: not so much complicated actually (see my answer).
    $endgroup$
    – G Cab
    Dec 30 '18 at 19:07
















4












$begingroup$


Let $n$ be an integer $geq 1.$ Then a partition of $n$ is a sequence of positive integers (greater than equal to $1$) such that their sum equals $n.$ So for instance if $n=4$ then
$$[[4], [1, 3], [1, 1, 2], [1, 1, 1, 1], [1, 2, 1], [2, 2], [2, 1, 1], [3, 1]]$$
is a collection of all partitions of $n$ with order. Clearly for any $n$ the number of such ordered partitions is $2^{n-1}.$ However I want to count the number of paritions of $n$ where each integer in the parition is less than equal to some integer $k.$ So for instance if $n=4$ and $k=2$ then
$$[[1, 1, 2], [1, 1, 1, 1], [1, 2, 1], [2, 2], [2, 1, 1]]$$
is a collection of all the $2$-partitions of $4.$ Is there a general formula for finding this or maybe an asymptotic expression?










share|cite|improve this question











$endgroup$












  • $begingroup$
    See this, specifically $A$-restricted compositions.
    $endgroup$
    – MathematicsStudent1122
    Dec 28 '18 at 21:48












  • $begingroup$
    Well, for $k=2$ you get the Fibonacci sequence, so if there is a general formula, it will be complicated for sure.
    $endgroup$
    – SmileyCraft
    Dec 28 '18 at 21:48










  • $begingroup$
    @MathematicsStudent1122 I saw this, but it does not give me the answer to my question. Maybe I am missing something. Perhaps you could elaborate?
    $endgroup$
    – model_checker
    Dec 28 '18 at 21:50










  • $begingroup$
    You can convert them into eqs. and find no. of integral solutions here is a link of finding no. of possible integral solutions math.stackexchange.com/a/2990467/584828
    $endgroup$
    – Mustang
    Dec 28 '18 at 21:52










  • $begingroup$
    @SmileyCraft: not so much complicated actually (see my answer).
    $endgroup$
    – G Cab
    Dec 30 '18 at 19:07














4












4








4


1



$begingroup$


Let $n$ be an integer $geq 1.$ Then a partition of $n$ is a sequence of positive integers (greater than equal to $1$) such that their sum equals $n.$ So for instance if $n=4$ then
$$[[4], [1, 3], [1, 1, 2], [1, 1, 1, 1], [1, 2, 1], [2, 2], [2, 1, 1], [3, 1]]$$
is a collection of all partitions of $n$ with order. Clearly for any $n$ the number of such ordered partitions is $2^{n-1}.$ However I want to count the number of paritions of $n$ where each integer in the parition is less than equal to some integer $k.$ So for instance if $n=4$ and $k=2$ then
$$[[1, 1, 2], [1, 1, 1, 1], [1, 2, 1], [2, 2], [2, 1, 1]]$$
is a collection of all the $2$-partitions of $4.$ Is there a general formula for finding this or maybe an asymptotic expression?










share|cite|improve this question











$endgroup$




Let $n$ be an integer $geq 1.$ Then a partition of $n$ is a sequence of positive integers (greater than equal to $1$) such that their sum equals $n.$ So for instance if $n=4$ then
$$[[4], [1, 3], [1, 1, 2], [1, 1, 1, 1], [1, 2, 1], [2, 2], [2, 1, 1], [3, 1]]$$
is a collection of all partitions of $n$ with order. Clearly for any $n$ the number of such ordered partitions is $2^{n-1}.$ However I want to count the number of paritions of $n$ where each integer in the parition is less than equal to some integer $k.$ So for instance if $n=4$ and $k=2$ then
$$[[1, 1, 2], [1, 1, 1, 1], [1, 2, 1], [2, 2], [2, 1, 1]]$$
is a collection of all the $2$-partitions of $4.$ Is there a general formula for finding this or maybe an asymptotic expression?







combinatorics number-theory asymptotics






share|cite|improve this question















share|cite|improve this question













share|cite|improve this question




share|cite|improve this question








edited Jan 3 at 3:51







model_checker

















asked Dec 28 '18 at 21:40









model_checkermodel_checker

4,45521931




4,45521931












  • $begingroup$
    See this, specifically $A$-restricted compositions.
    $endgroup$
    – MathematicsStudent1122
    Dec 28 '18 at 21:48












  • $begingroup$
    Well, for $k=2$ you get the Fibonacci sequence, so if there is a general formula, it will be complicated for sure.
    $endgroup$
    – SmileyCraft
    Dec 28 '18 at 21:48










  • $begingroup$
    @MathematicsStudent1122 I saw this, but it does not give me the answer to my question. Maybe I am missing something. Perhaps you could elaborate?
    $endgroup$
    – model_checker
    Dec 28 '18 at 21:50










  • $begingroup$
    You can convert them into eqs. and find no. of integral solutions here is a link of finding no. of possible integral solutions math.stackexchange.com/a/2990467/584828
    $endgroup$
    – Mustang
    Dec 28 '18 at 21:52










  • $begingroup$
    @SmileyCraft: not so much complicated actually (see my answer).
    $endgroup$
    – G Cab
    Dec 30 '18 at 19:07


















  • $begingroup$
    See this, specifically $A$-restricted compositions.
    $endgroup$
    – MathematicsStudent1122
    Dec 28 '18 at 21:48












  • $begingroup$
    Well, for $k=2$ you get the Fibonacci sequence, so if there is a general formula, it will be complicated for sure.
    $endgroup$
    – SmileyCraft
    Dec 28 '18 at 21:48










  • $begingroup$
    @MathematicsStudent1122 I saw this, but it does not give me the answer to my question. Maybe I am missing something. Perhaps you could elaborate?
    $endgroup$
    – model_checker
    Dec 28 '18 at 21:50










  • $begingroup$
    You can convert them into eqs. and find no. of integral solutions here is a link of finding no. of possible integral solutions math.stackexchange.com/a/2990467/584828
    $endgroup$
    – Mustang
    Dec 28 '18 at 21:52










  • $begingroup$
    @SmileyCraft: not so much complicated actually (see my answer).
    $endgroup$
    – G Cab
    Dec 30 '18 at 19:07
















$begingroup$
See this, specifically $A$-restricted compositions.
$endgroup$
– MathematicsStudent1122
Dec 28 '18 at 21:48






$begingroup$
See this, specifically $A$-restricted compositions.
$endgroup$
– MathematicsStudent1122
Dec 28 '18 at 21:48














$begingroup$
Well, for $k=2$ you get the Fibonacci sequence, so if there is a general formula, it will be complicated for sure.
$endgroup$
– SmileyCraft
Dec 28 '18 at 21:48




$begingroup$
Well, for $k=2$ you get the Fibonacci sequence, so if there is a general formula, it will be complicated for sure.
$endgroup$
– SmileyCraft
Dec 28 '18 at 21:48












$begingroup$
@MathematicsStudent1122 I saw this, but it does not give me the answer to my question. Maybe I am missing something. Perhaps you could elaborate?
$endgroup$
– model_checker
Dec 28 '18 at 21:50




$begingroup$
@MathematicsStudent1122 I saw this, but it does not give me the answer to my question. Maybe I am missing something. Perhaps you could elaborate?
$endgroup$
– model_checker
Dec 28 '18 at 21:50












$begingroup$
You can convert them into eqs. and find no. of integral solutions here is a link of finding no. of possible integral solutions math.stackexchange.com/a/2990467/584828
$endgroup$
– Mustang
Dec 28 '18 at 21:52




$begingroup$
You can convert them into eqs. and find no. of integral solutions here is a link of finding no. of possible integral solutions math.stackexchange.com/a/2990467/584828
$endgroup$
– Mustang
Dec 28 '18 at 21:52












$begingroup$
@SmileyCraft: not so much complicated actually (see my answer).
$endgroup$
– G Cab
Dec 30 '18 at 19:07




$begingroup$
@SmileyCraft: not so much complicated actually (see my answer).
$endgroup$
– G Cab
Dec 30 '18 at 19:07










3 Answers
3






active

oldest

votes


















2












$begingroup$

I think it may be easier to deal with the simpler case of $k=2$ first. Let's find some easy, small values first:



$$n=1rightarrow [[1]]rightarrow f(1)=1$$
$$n=2rightarrow [[1,1],[2]]rightarrow f(2)=2$$
$$n=3rightarrow [[1,1,1],[2,1],[1,2]]rightarrow f(3)=3$$



As you have shown, $f(4)=5$.



$$n=5rightarrow [[1,1,2,1],[1,1,1,1,1],[1,2,1,1],[2,2,1],[1,1,2,1],[1,1,1,2],[2,1,2],[1,2,2]]rightarrow f(5)=8$$



Hopefully you notice the pattern: This is the Fibonacci sequence. The reason this is the Fibonacci sequence is because for any $n$, a partition with $k=2$ will end in either $1$ or $2$. If it ends in $1$, then we simply add $1$ onto a partition from $n-1$. If it ends in $2$, then we simply add $2$ onto a partition from $n-2$. This gives us the following recurrence:



$$f(n)=f(n-1)+f(n-2)$$



And, of course, this is the Fibonacci recurrence.



Now, let's extend this reasoning to general $k$. First, let's define $f(n, k)$ to be the number of partitions of $N$ where the positive integers involved are at most $k$. Then, let's define $f(n,k)=0$ for any $n < 0$ because we only want to allow for positive integers and $f(0,k)=1$ for any $k$ since the only way to partition $0$ is with the empty set.



Since the integers must be at most $k$, this means in every partition, we are adding some new integer $l leq k$ every time. Thus, if we take $l$ away, we get a partition of $n-l$. Therefore, the number of partitions of $n$ must be the partitions of $n-1$ plus that of $n-2$ plus that of $n-3$ ... and so on, until $n-k$. In other words:



$$f(n,k)=sum_{l=1}^k f(n-l,k)$$



This is a generalization of the Fibonacci numbers. For $k=3$, it gets you the tribonacci numbers, for $k=4$, if gets you the tetranacci numbers, then $k=5$ is pentanacci, $k=6$ is heptanacci, etc.



Thus, the pattern you are studying is simply a higher-order form of the Fibonacci numbers.






share|cite|improve this answer









$endgroup$









  • 1




    $begingroup$
    For computing the $k$'th order Fibonacci numbers, you can use the formula $f(n,k)=2f(n-1,k)-f(n-k-1,k)$.
    $endgroup$
    – SmileyCraft
    Dec 28 '18 at 22:00










  • $begingroup$
    For a given $k,$ the initial conditions for the recurrence are $f(1,k) = 1, f(2,k) = 2, f(3,k) = 2^{3-1}=4, cdots f(k,k)=2^{k-1},$ right?
    $endgroup$
    – model_checker
    Dec 29 '18 at 19:04












  • $begingroup$
    @Hello_World No, I defined the initial conditions as $f(n,k)=0$ for $n < 0$ and $f(0,k)=1$ for $n=0$. Then, you can figure out $f(1,k), f(2,k), ...$ from the initial conditions using the recurrence.
    $endgroup$
    – Noble Mushtak
    Dec 29 '18 at 19:52



















2












$begingroup$

Generating functions look like the way to go. Finding $m$ pieces, each of size at most $k$, that sum to $n$, has generating function $(x+x^2+cdots+x^k)^m$; the coefficient of $x^n$ is the number of ways. Then, we want the total number of ways, over all possible numbers of pieces, so we get
begin{align*}F(x) &= sum_{m=0}^{infty} (x+x^2+cdots+x^k)^m\
&= frac{1}{1-(x+x^2+cdots+x^k)}\
F(x) &= frac{1}{1-frac{x-x^{k+1}}{1-x}} = frac{1-x}{1-2x+x^{k+1}}end{align*}

If you're looking for specific coefficients, it's easiest to use the Fibonacci-like recursion noted in Noble Mushtak's answer, or the telescoped version $a_{n+1}=2a_n-a_{n-k}$. The initial conditions are $a_0=a_1=1,a_{-1}=cdots=a_{-k+1}=0$. For the asymptotics? That'll come from the pole of $F$ nearest to zero - namely, the unique positive root $r$ of $x^k+cdots+x^2+x-1=0$. For $k>1$, $rin [0,1]$. The $n$th term will be approximately proportional to $r^{-n}$. An exact formula? You'd have to find all the roots of that polynomial and run partial fractions. The generating function then splits up as several terms of the form $frac{b_j}{x-c_j}$, each of which gives us a geometric series. It's all fine in theory, but that first step of solving the polynomial is a doozy.






share|cite|improve this answer









$endgroup$













  • $begingroup$
    you might be interested to know that the coefficients of $F(x)$ might be expressed through a finite sum (see my answer).
    $endgroup$
    – G Cab
    Dec 30 '18 at 19:01



















1












$begingroup$

First a note about terminology:

- a Partition of a positive integer $n$ is a non-decreasing sequence of positive integers summing to $n$;

- a Composition of a positive integer $n$ is an unordered sequence of positive integers summing to $n$.



That premised, you are speaking of the number of Compositions of $n$, whose terms (parts) are not-greater than $k$.



Consider the case in which you seek for the number of compositions of the positive number $s+m$
into $m$ parts not exceeding $r+1$
$$
{rm No}{rm .},{rm of},{rm solutions},{rm to};left{ matrix{
{rm 1} le {rm integer};y_{,j} le r + 1 hfill cr
y_{,1} + y_{,2} + ; cdots ; + y_{,m} = s + m hfill cr} right.
$$

that's the same as
$$N_{,b} (s,r,m) = text{No}text{. of solutions to};left{ begin{gathered}
0 leqslant text{integer }x_{,j} leqslant r hfill \
x_{,1} + x_{,2} + cdots + x_{,m} = s hfill \
end{gathered} right.$$



$N_b$ is given by the following sum
$$
N_b (s,r,m)quad left| {;0 leqslant text{integers }s,m,r} right.quad =
sumlimits_{left( {0, leqslant } right),,j,,left( { leqslant ,frac{s}{r+1}, leqslant ,m} right)}
{left( { - 1} right)^k binom{m}{j}
binom
{ s + m - 1 - jleft( {r + 1} right) }
{ s - jleft( {r + 1} right)} }
$$

as widely explained in this related post.



Going back to your case, the
number of compositions of $n$ into $m$ parts not greater than $k$

will then be
$$
eqalign{
& N_c (n,k,m)quad left| {;1 le n,m,k} right.quad = cr
& = sumlimits_{left( {0, le } right),,j,,left( {, le ,m} right)} {left( { - 1} right)^{,j} binom{m}{j}
binom{n - 1 - j,k}{ n - m - j,k}} cr}
$$

while the
overall number of compositions of $n$ into parts not greater than $k$

will be the sum
of the above for $0 le m$ : if the sum extends over $n$ it will maintain the value $2^{n-1}$.
$$ bbox[lightyellow] {
eqalign{
& N_{c,t} (n,k)quad left| {;0 le n,k in mathbb Z} right.quad = cr
& = sumlimits_{0, le ,,m} {;sumlimits_{left( {0, le } right),,j,,left( {, le ,m} right)} {left( { - 1} right)^{,j}
binom{m}{j} binom{ n - 1 - j,k}{n - m - j,k}
} } cr}
}$$



In the example with $n=4$, the above gives for $k=1,2,3,4$
$$
1,5,7,8
$$

which in fact correspond to
$$
eqalign{
& left[ {1,1,1,1} right] cr
& left[ {1,1,1,1} right]left[ {1,1,2} right]left[ {1,2,1} right]left[ {2,1,1} right]left[ {2,2} right] cr
& left[ {1,1,1,1} right]left[ {1,1,2} right]left[ {1,2,1} right]left[ {2,1,1} right]left[ {2,2} right]left[ {1,3} right]left[ {3,1} right] cr
& left[ {1,1,1,1} right]left[ {1,1,2} right]left[ {1,2,1} right]left[ {2,1,1} right]left[ {2,2} right]left[ {1,3} right]left[ {3,1} right]left[ 4 right] cr}
$$



Finally note that:

- $N_{c,t}(n,k)$ correctly checks with OEIS seq. A126198, which provides further properties of these numbers;

- the o.g.f. of $N_{c,t}$ in $n$ is in fact the $F(x)$ given by @jmerry's answer (re. to the o.g.f. for $N_b$ given in related post);
$$
sumlimits_{0, le ,,n} {N_{c,t} (n,k),x^{,n} } = {{1 - x} over {1 - 2x + x^{,k + 1} }}
$$

- $N_{c,t}(n,k)$ satisfies the recursion given by @NobleMushtak
$$
N_{c,t} (n,k) = sumlimits_{j = 1}^k {N_{c,t} (n - j,k)} + left[ {n = 0} right]
$$

where $[P]$ denotes the Iverson bracket.






share|cite|improve this answer











$endgroup$














    Your Answer








    StackExchange.ready(function() {
    var channelOptions = {
    tags: "".split(" "),
    id: "69"
    };
    initTagRenderer("".split(" "), "".split(" "), channelOptions);

    StackExchange.using("externalEditor", function() {
    // Have to fire editor after snippets, if snippets enabled
    if (StackExchange.settings.snippets.snippetsEnabled) {
    StackExchange.using("snippets", function() {
    createEditor();
    });
    }
    else {
    createEditor();
    }
    });

    function createEditor() {
    StackExchange.prepareEditor({
    heartbeatType: 'answer',
    autoActivateHeartbeat: false,
    convertImagesToLinks: true,
    noModals: true,
    showLowRepImageUploadWarning: true,
    reputationToPostImages: 10,
    bindNavPrevention: true,
    postfix: "",
    imageUploader: {
    brandingHtml: "Powered by u003ca class="icon-imgur-white" href="https://imgur.com/"u003eu003c/au003e",
    contentPolicyHtml: "User contributions licensed under u003ca href="https://creativecommons.org/licenses/by-sa/3.0/"u003ecc by-sa 3.0 with attribution requiredu003c/au003e u003ca href="https://stackoverflow.com/legal/content-policy"u003e(content policy)u003c/au003e",
    allowUrls: true
    },
    noCode: true, onDemand: true,
    discardSelector: ".discard-answer"
    ,immediatelyShowMarkdownHelp:true
    });


    }
    });














    draft saved

    draft discarded


















    StackExchange.ready(
    function () {
    StackExchange.openid.initPostLogin('.new-post-login', 'https%3a%2f%2fmath.stackexchange.com%2fquestions%2f3055314%2fnumber-of-compositions-of-n-such-that-each-term-is-less-than-equal-to-k%23new-answer', 'question_page');
    }
    );

    Post as a guest















    Required, but never shown

























    3 Answers
    3






    active

    oldest

    votes








    3 Answers
    3






    active

    oldest

    votes









    active

    oldest

    votes






    active

    oldest

    votes









    2












    $begingroup$

    I think it may be easier to deal with the simpler case of $k=2$ first. Let's find some easy, small values first:



    $$n=1rightarrow [[1]]rightarrow f(1)=1$$
    $$n=2rightarrow [[1,1],[2]]rightarrow f(2)=2$$
    $$n=3rightarrow [[1,1,1],[2,1],[1,2]]rightarrow f(3)=3$$



    As you have shown, $f(4)=5$.



    $$n=5rightarrow [[1,1,2,1],[1,1,1,1,1],[1,2,1,1],[2,2,1],[1,1,2,1],[1,1,1,2],[2,1,2],[1,2,2]]rightarrow f(5)=8$$



    Hopefully you notice the pattern: This is the Fibonacci sequence. The reason this is the Fibonacci sequence is because for any $n$, a partition with $k=2$ will end in either $1$ or $2$. If it ends in $1$, then we simply add $1$ onto a partition from $n-1$. If it ends in $2$, then we simply add $2$ onto a partition from $n-2$. This gives us the following recurrence:



    $$f(n)=f(n-1)+f(n-2)$$



    And, of course, this is the Fibonacci recurrence.



    Now, let's extend this reasoning to general $k$. First, let's define $f(n, k)$ to be the number of partitions of $N$ where the positive integers involved are at most $k$. Then, let's define $f(n,k)=0$ for any $n < 0$ because we only want to allow for positive integers and $f(0,k)=1$ for any $k$ since the only way to partition $0$ is with the empty set.



    Since the integers must be at most $k$, this means in every partition, we are adding some new integer $l leq k$ every time. Thus, if we take $l$ away, we get a partition of $n-l$. Therefore, the number of partitions of $n$ must be the partitions of $n-1$ plus that of $n-2$ plus that of $n-3$ ... and so on, until $n-k$. In other words:



    $$f(n,k)=sum_{l=1}^k f(n-l,k)$$



    This is a generalization of the Fibonacci numbers. For $k=3$, it gets you the tribonacci numbers, for $k=4$, if gets you the tetranacci numbers, then $k=5$ is pentanacci, $k=6$ is heptanacci, etc.



    Thus, the pattern you are studying is simply a higher-order form of the Fibonacci numbers.






    share|cite|improve this answer









    $endgroup$









    • 1




      $begingroup$
      For computing the $k$'th order Fibonacci numbers, you can use the formula $f(n,k)=2f(n-1,k)-f(n-k-1,k)$.
      $endgroup$
      – SmileyCraft
      Dec 28 '18 at 22:00










    • $begingroup$
      For a given $k,$ the initial conditions for the recurrence are $f(1,k) = 1, f(2,k) = 2, f(3,k) = 2^{3-1}=4, cdots f(k,k)=2^{k-1},$ right?
      $endgroup$
      – model_checker
      Dec 29 '18 at 19:04












    • $begingroup$
      @Hello_World No, I defined the initial conditions as $f(n,k)=0$ for $n < 0$ and $f(0,k)=1$ for $n=0$. Then, you can figure out $f(1,k), f(2,k), ...$ from the initial conditions using the recurrence.
      $endgroup$
      – Noble Mushtak
      Dec 29 '18 at 19:52
















    2












    $begingroup$

    I think it may be easier to deal with the simpler case of $k=2$ first. Let's find some easy, small values first:



    $$n=1rightarrow [[1]]rightarrow f(1)=1$$
    $$n=2rightarrow [[1,1],[2]]rightarrow f(2)=2$$
    $$n=3rightarrow [[1,1,1],[2,1],[1,2]]rightarrow f(3)=3$$



    As you have shown, $f(4)=5$.



    $$n=5rightarrow [[1,1,2,1],[1,1,1,1,1],[1,2,1,1],[2,2,1],[1,1,2,1],[1,1,1,2],[2,1,2],[1,2,2]]rightarrow f(5)=8$$



    Hopefully you notice the pattern: This is the Fibonacci sequence. The reason this is the Fibonacci sequence is because for any $n$, a partition with $k=2$ will end in either $1$ or $2$. If it ends in $1$, then we simply add $1$ onto a partition from $n-1$. If it ends in $2$, then we simply add $2$ onto a partition from $n-2$. This gives us the following recurrence:



    $$f(n)=f(n-1)+f(n-2)$$



    And, of course, this is the Fibonacci recurrence.



    Now, let's extend this reasoning to general $k$. First, let's define $f(n, k)$ to be the number of partitions of $N$ where the positive integers involved are at most $k$. Then, let's define $f(n,k)=0$ for any $n < 0$ because we only want to allow for positive integers and $f(0,k)=1$ for any $k$ since the only way to partition $0$ is with the empty set.



    Since the integers must be at most $k$, this means in every partition, we are adding some new integer $l leq k$ every time. Thus, if we take $l$ away, we get a partition of $n-l$. Therefore, the number of partitions of $n$ must be the partitions of $n-1$ plus that of $n-2$ plus that of $n-3$ ... and so on, until $n-k$. In other words:



    $$f(n,k)=sum_{l=1}^k f(n-l,k)$$



    This is a generalization of the Fibonacci numbers. For $k=3$, it gets you the tribonacci numbers, for $k=4$, if gets you the tetranacci numbers, then $k=5$ is pentanacci, $k=6$ is heptanacci, etc.



    Thus, the pattern you are studying is simply a higher-order form of the Fibonacci numbers.






    share|cite|improve this answer









    $endgroup$









    • 1




      $begingroup$
      For computing the $k$'th order Fibonacci numbers, you can use the formula $f(n,k)=2f(n-1,k)-f(n-k-1,k)$.
      $endgroup$
      – SmileyCraft
      Dec 28 '18 at 22:00










    • $begingroup$
      For a given $k,$ the initial conditions for the recurrence are $f(1,k) = 1, f(2,k) = 2, f(3,k) = 2^{3-1}=4, cdots f(k,k)=2^{k-1},$ right?
      $endgroup$
      – model_checker
      Dec 29 '18 at 19:04












    • $begingroup$
      @Hello_World No, I defined the initial conditions as $f(n,k)=0$ for $n < 0$ and $f(0,k)=1$ for $n=0$. Then, you can figure out $f(1,k), f(2,k), ...$ from the initial conditions using the recurrence.
      $endgroup$
      – Noble Mushtak
      Dec 29 '18 at 19:52














    2












    2








    2





    $begingroup$

    I think it may be easier to deal with the simpler case of $k=2$ first. Let's find some easy, small values first:



    $$n=1rightarrow [[1]]rightarrow f(1)=1$$
    $$n=2rightarrow [[1,1],[2]]rightarrow f(2)=2$$
    $$n=3rightarrow [[1,1,1],[2,1],[1,2]]rightarrow f(3)=3$$



    As you have shown, $f(4)=5$.



    $$n=5rightarrow [[1,1,2,1],[1,1,1,1,1],[1,2,1,1],[2,2,1],[1,1,2,1],[1,1,1,2],[2,1,2],[1,2,2]]rightarrow f(5)=8$$



    Hopefully you notice the pattern: This is the Fibonacci sequence. The reason this is the Fibonacci sequence is because for any $n$, a partition with $k=2$ will end in either $1$ or $2$. If it ends in $1$, then we simply add $1$ onto a partition from $n-1$. If it ends in $2$, then we simply add $2$ onto a partition from $n-2$. This gives us the following recurrence:



    $$f(n)=f(n-1)+f(n-2)$$



    And, of course, this is the Fibonacci recurrence.



    Now, let's extend this reasoning to general $k$. First, let's define $f(n, k)$ to be the number of partitions of $N$ where the positive integers involved are at most $k$. Then, let's define $f(n,k)=0$ for any $n < 0$ because we only want to allow for positive integers and $f(0,k)=1$ for any $k$ since the only way to partition $0$ is with the empty set.



    Since the integers must be at most $k$, this means in every partition, we are adding some new integer $l leq k$ every time. Thus, if we take $l$ away, we get a partition of $n-l$. Therefore, the number of partitions of $n$ must be the partitions of $n-1$ plus that of $n-2$ plus that of $n-3$ ... and so on, until $n-k$. In other words:



    $$f(n,k)=sum_{l=1}^k f(n-l,k)$$



    This is a generalization of the Fibonacci numbers. For $k=3$, it gets you the tribonacci numbers, for $k=4$, if gets you the tetranacci numbers, then $k=5$ is pentanacci, $k=6$ is heptanacci, etc.



    Thus, the pattern you are studying is simply a higher-order form of the Fibonacci numbers.






    share|cite|improve this answer









    $endgroup$



    I think it may be easier to deal with the simpler case of $k=2$ first. Let's find some easy, small values first:



    $$n=1rightarrow [[1]]rightarrow f(1)=1$$
    $$n=2rightarrow [[1,1],[2]]rightarrow f(2)=2$$
    $$n=3rightarrow [[1,1,1],[2,1],[1,2]]rightarrow f(3)=3$$



    As you have shown, $f(4)=5$.



    $$n=5rightarrow [[1,1,2,1],[1,1,1,1,1],[1,2,1,1],[2,2,1],[1,1,2,1],[1,1,1,2],[2,1,2],[1,2,2]]rightarrow f(5)=8$$



    Hopefully you notice the pattern: This is the Fibonacci sequence. The reason this is the Fibonacci sequence is because for any $n$, a partition with $k=2$ will end in either $1$ or $2$. If it ends in $1$, then we simply add $1$ onto a partition from $n-1$. If it ends in $2$, then we simply add $2$ onto a partition from $n-2$. This gives us the following recurrence:



    $$f(n)=f(n-1)+f(n-2)$$



    And, of course, this is the Fibonacci recurrence.



    Now, let's extend this reasoning to general $k$. First, let's define $f(n, k)$ to be the number of partitions of $N$ where the positive integers involved are at most $k$. Then, let's define $f(n,k)=0$ for any $n < 0$ because we only want to allow for positive integers and $f(0,k)=1$ for any $k$ since the only way to partition $0$ is with the empty set.



    Since the integers must be at most $k$, this means in every partition, we are adding some new integer $l leq k$ every time. Thus, if we take $l$ away, we get a partition of $n-l$. Therefore, the number of partitions of $n$ must be the partitions of $n-1$ plus that of $n-2$ plus that of $n-3$ ... and so on, until $n-k$. In other words:



    $$f(n,k)=sum_{l=1}^k f(n-l,k)$$



    This is a generalization of the Fibonacci numbers. For $k=3$, it gets you the tribonacci numbers, for $k=4$, if gets you the tetranacci numbers, then $k=5$ is pentanacci, $k=6$ is heptanacci, etc.



    Thus, the pattern you are studying is simply a higher-order form of the Fibonacci numbers.







    share|cite|improve this answer












    share|cite|improve this answer



    share|cite|improve this answer










    answered Dec 28 '18 at 21:56









    Noble MushtakNoble Mushtak

    15.4k1835




    15.4k1835








    • 1




      $begingroup$
      For computing the $k$'th order Fibonacci numbers, you can use the formula $f(n,k)=2f(n-1,k)-f(n-k-1,k)$.
      $endgroup$
      – SmileyCraft
      Dec 28 '18 at 22:00










    • $begingroup$
      For a given $k,$ the initial conditions for the recurrence are $f(1,k) = 1, f(2,k) = 2, f(3,k) = 2^{3-1}=4, cdots f(k,k)=2^{k-1},$ right?
      $endgroup$
      – model_checker
      Dec 29 '18 at 19:04












    • $begingroup$
      @Hello_World No, I defined the initial conditions as $f(n,k)=0$ for $n < 0$ and $f(0,k)=1$ for $n=0$. Then, you can figure out $f(1,k), f(2,k), ...$ from the initial conditions using the recurrence.
      $endgroup$
      – Noble Mushtak
      Dec 29 '18 at 19:52














    • 1




      $begingroup$
      For computing the $k$'th order Fibonacci numbers, you can use the formula $f(n,k)=2f(n-1,k)-f(n-k-1,k)$.
      $endgroup$
      – SmileyCraft
      Dec 28 '18 at 22:00










    • $begingroup$
      For a given $k,$ the initial conditions for the recurrence are $f(1,k) = 1, f(2,k) = 2, f(3,k) = 2^{3-1}=4, cdots f(k,k)=2^{k-1},$ right?
      $endgroup$
      – model_checker
      Dec 29 '18 at 19:04












    • $begingroup$
      @Hello_World No, I defined the initial conditions as $f(n,k)=0$ for $n < 0$ and $f(0,k)=1$ for $n=0$. Then, you can figure out $f(1,k), f(2,k), ...$ from the initial conditions using the recurrence.
      $endgroup$
      – Noble Mushtak
      Dec 29 '18 at 19:52








    1




    1




    $begingroup$
    For computing the $k$'th order Fibonacci numbers, you can use the formula $f(n,k)=2f(n-1,k)-f(n-k-1,k)$.
    $endgroup$
    – SmileyCraft
    Dec 28 '18 at 22:00




    $begingroup$
    For computing the $k$'th order Fibonacci numbers, you can use the formula $f(n,k)=2f(n-1,k)-f(n-k-1,k)$.
    $endgroup$
    – SmileyCraft
    Dec 28 '18 at 22:00












    $begingroup$
    For a given $k,$ the initial conditions for the recurrence are $f(1,k) = 1, f(2,k) = 2, f(3,k) = 2^{3-1}=4, cdots f(k,k)=2^{k-1},$ right?
    $endgroup$
    – model_checker
    Dec 29 '18 at 19:04






    $begingroup$
    For a given $k,$ the initial conditions for the recurrence are $f(1,k) = 1, f(2,k) = 2, f(3,k) = 2^{3-1}=4, cdots f(k,k)=2^{k-1},$ right?
    $endgroup$
    – model_checker
    Dec 29 '18 at 19:04














    $begingroup$
    @Hello_World No, I defined the initial conditions as $f(n,k)=0$ for $n < 0$ and $f(0,k)=1$ for $n=0$. Then, you can figure out $f(1,k), f(2,k), ...$ from the initial conditions using the recurrence.
    $endgroup$
    – Noble Mushtak
    Dec 29 '18 at 19:52




    $begingroup$
    @Hello_World No, I defined the initial conditions as $f(n,k)=0$ for $n < 0$ and $f(0,k)=1$ for $n=0$. Then, you can figure out $f(1,k), f(2,k), ...$ from the initial conditions using the recurrence.
    $endgroup$
    – Noble Mushtak
    Dec 29 '18 at 19:52











    2












    $begingroup$

    Generating functions look like the way to go. Finding $m$ pieces, each of size at most $k$, that sum to $n$, has generating function $(x+x^2+cdots+x^k)^m$; the coefficient of $x^n$ is the number of ways. Then, we want the total number of ways, over all possible numbers of pieces, so we get
    begin{align*}F(x) &= sum_{m=0}^{infty} (x+x^2+cdots+x^k)^m\
    &= frac{1}{1-(x+x^2+cdots+x^k)}\
    F(x) &= frac{1}{1-frac{x-x^{k+1}}{1-x}} = frac{1-x}{1-2x+x^{k+1}}end{align*}

    If you're looking for specific coefficients, it's easiest to use the Fibonacci-like recursion noted in Noble Mushtak's answer, or the telescoped version $a_{n+1}=2a_n-a_{n-k}$. The initial conditions are $a_0=a_1=1,a_{-1}=cdots=a_{-k+1}=0$. For the asymptotics? That'll come from the pole of $F$ nearest to zero - namely, the unique positive root $r$ of $x^k+cdots+x^2+x-1=0$. For $k>1$, $rin [0,1]$. The $n$th term will be approximately proportional to $r^{-n}$. An exact formula? You'd have to find all the roots of that polynomial and run partial fractions. The generating function then splits up as several terms of the form $frac{b_j}{x-c_j}$, each of which gives us a geometric series. It's all fine in theory, but that first step of solving the polynomial is a doozy.






    share|cite|improve this answer









    $endgroup$













    • $begingroup$
      you might be interested to know that the coefficients of $F(x)$ might be expressed through a finite sum (see my answer).
      $endgroup$
      – G Cab
      Dec 30 '18 at 19:01
















    2












    $begingroup$

    Generating functions look like the way to go. Finding $m$ pieces, each of size at most $k$, that sum to $n$, has generating function $(x+x^2+cdots+x^k)^m$; the coefficient of $x^n$ is the number of ways. Then, we want the total number of ways, over all possible numbers of pieces, so we get
    begin{align*}F(x) &= sum_{m=0}^{infty} (x+x^2+cdots+x^k)^m\
    &= frac{1}{1-(x+x^2+cdots+x^k)}\
    F(x) &= frac{1}{1-frac{x-x^{k+1}}{1-x}} = frac{1-x}{1-2x+x^{k+1}}end{align*}

    If you're looking for specific coefficients, it's easiest to use the Fibonacci-like recursion noted in Noble Mushtak's answer, or the telescoped version $a_{n+1}=2a_n-a_{n-k}$. The initial conditions are $a_0=a_1=1,a_{-1}=cdots=a_{-k+1}=0$. For the asymptotics? That'll come from the pole of $F$ nearest to zero - namely, the unique positive root $r$ of $x^k+cdots+x^2+x-1=0$. For $k>1$, $rin [0,1]$. The $n$th term will be approximately proportional to $r^{-n}$. An exact formula? You'd have to find all the roots of that polynomial and run partial fractions. The generating function then splits up as several terms of the form $frac{b_j}{x-c_j}$, each of which gives us a geometric series. It's all fine in theory, but that first step of solving the polynomial is a doozy.






    share|cite|improve this answer









    $endgroup$













    • $begingroup$
      you might be interested to know that the coefficients of $F(x)$ might be expressed through a finite sum (see my answer).
      $endgroup$
      – G Cab
      Dec 30 '18 at 19:01














    2












    2








    2





    $begingroup$

    Generating functions look like the way to go. Finding $m$ pieces, each of size at most $k$, that sum to $n$, has generating function $(x+x^2+cdots+x^k)^m$; the coefficient of $x^n$ is the number of ways. Then, we want the total number of ways, over all possible numbers of pieces, so we get
    begin{align*}F(x) &= sum_{m=0}^{infty} (x+x^2+cdots+x^k)^m\
    &= frac{1}{1-(x+x^2+cdots+x^k)}\
    F(x) &= frac{1}{1-frac{x-x^{k+1}}{1-x}} = frac{1-x}{1-2x+x^{k+1}}end{align*}

    If you're looking for specific coefficients, it's easiest to use the Fibonacci-like recursion noted in Noble Mushtak's answer, or the telescoped version $a_{n+1}=2a_n-a_{n-k}$. The initial conditions are $a_0=a_1=1,a_{-1}=cdots=a_{-k+1}=0$. For the asymptotics? That'll come from the pole of $F$ nearest to zero - namely, the unique positive root $r$ of $x^k+cdots+x^2+x-1=0$. For $k>1$, $rin [0,1]$. The $n$th term will be approximately proportional to $r^{-n}$. An exact formula? You'd have to find all the roots of that polynomial and run partial fractions. The generating function then splits up as several terms of the form $frac{b_j}{x-c_j}$, each of which gives us a geometric series. It's all fine in theory, but that first step of solving the polynomial is a doozy.






    share|cite|improve this answer









    $endgroup$



    Generating functions look like the way to go. Finding $m$ pieces, each of size at most $k$, that sum to $n$, has generating function $(x+x^2+cdots+x^k)^m$; the coefficient of $x^n$ is the number of ways. Then, we want the total number of ways, over all possible numbers of pieces, so we get
    begin{align*}F(x) &= sum_{m=0}^{infty} (x+x^2+cdots+x^k)^m\
    &= frac{1}{1-(x+x^2+cdots+x^k)}\
    F(x) &= frac{1}{1-frac{x-x^{k+1}}{1-x}} = frac{1-x}{1-2x+x^{k+1}}end{align*}

    If you're looking for specific coefficients, it's easiest to use the Fibonacci-like recursion noted in Noble Mushtak's answer, or the telescoped version $a_{n+1}=2a_n-a_{n-k}$. The initial conditions are $a_0=a_1=1,a_{-1}=cdots=a_{-k+1}=0$. For the asymptotics? That'll come from the pole of $F$ nearest to zero - namely, the unique positive root $r$ of $x^k+cdots+x^2+x-1=0$. For $k>1$, $rin [0,1]$. The $n$th term will be approximately proportional to $r^{-n}$. An exact formula? You'd have to find all the roots of that polynomial and run partial fractions. The generating function then splits up as several terms of the form $frac{b_j}{x-c_j}$, each of which gives us a geometric series. It's all fine in theory, but that first step of solving the polynomial is a doozy.







    share|cite|improve this answer












    share|cite|improve this answer



    share|cite|improve this answer










    answered Dec 28 '18 at 22:15









    jmerryjmerry

    17.1k11633




    17.1k11633












    • $begingroup$
      you might be interested to know that the coefficients of $F(x)$ might be expressed through a finite sum (see my answer).
      $endgroup$
      – G Cab
      Dec 30 '18 at 19:01


















    • $begingroup$
      you might be interested to know that the coefficients of $F(x)$ might be expressed through a finite sum (see my answer).
      $endgroup$
      – G Cab
      Dec 30 '18 at 19:01
















    $begingroup$
    you might be interested to know that the coefficients of $F(x)$ might be expressed through a finite sum (see my answer).
    $endgroup$
    – G Cab
    Dec 30 '18 at 19:01




    $begingroup$
    you might be interested to know that the coefficients of $F(x)$ might be expressed through a finite sum (see my answer).
    $endgroup$
    – G Cab
    Dec 30 '18 at 19:01











    1












    $begingroup$

    First a note about terminology:

    - a Partition of a positive integer $n$ is a non-decreasing sequence of positive integers summing to $n$;

    - a Composition of a positive integer $n$ is an unordered sequence of positive integers summing to $n$.



    That premised, you are speaking of the number of Compositions of $n$, whose terms (parts) are not-greater than $k$.



    Consider the case in which you seek for the number of compositions of the positive number $s+m$
    into $m$ parts not exceeding $r+1$
    $$
    {rm No}{rm .},{rm of},{rm solutions},{rm to};left{ matrix{
    {rm 1} le {rm integer};y_{,j} le r + 1 hfill cr
    y_{,1} + y_{,2} + ; cdots ; + y_{,m} = s + m hfill cr} right.
    $$

    that's the same as
    $$N_{,b} (s,r,m) = text{No}text{. of solutions to};left{ begin{gathered}
    0 leqslant text{integer }x_{,j} leqslant r hfill \
    x_{,1} + x_{,2} + cdots + x_{,m} = s hfill \
    end{gathered} right.$$



    $N_b$ is given by the following sum
    $$
    N_b (s,r,m)quad left| {;0 leqslant text{integers }s,m,r} right.quad =
    sumlimits_{left( {0, leqslant } right),,j,,left( { leqslant ,frac{s}{r+1}, leqslant ,m} right)}
    {left( { - 1} right)^k binom{m}{j}
    binom
    { s + m - 1 - jleft( {r + 1} right) }
    { s - jleft( {r + 1} right)} }
    $$

    as widely explained in this related post.



    Going back to your case, the
    number of compositions of $n$ into $m$ parts not greater than $k$

    will then be
    $$
    eqalign{
    & N_c (n,k,m)quad left| {;1 le n,m,k} right.quad = cr
    & = sumlimits_{left( {0, le } right),,j,,left( {, le ,m} right)} {left( { - 1} right)^{,j} binom{m}{j}
    binom{n - 1 - j,k}{ n - m - j,k}} cr}
    $$

    while the
    overall number of compositions of $n$ into parts not greater than $k$

    will be the sum
    of the above for $0 le m$ : if the sum extends over $n$ it will maintain the value $2^{n-1}$.
    $$ bbox[lightyellow] {
    eqalign{
    & N_{c,t} (n,k)quad left| {;0 le n,k in mathbb Z} right.quad = cr
    & = sumlimits_{0, le ,,m} {;sumlimits_{left( {0, le } right),,j,,left( {, le ,m} right)} {left( { - 1} right)^{,j}
    binom{m}{j} binom{ n - 1 - j,k}{n - m - j,k}
    } } cr}
    }$$



    In the example with $n=4$, the above gives for $k=1,2,3,4$
    $$
    1,5,7,8
    $$

    which in fact correspond to
    $$
    eqalign{
    & left[ {1,1,1,1} right] cr
    & left[ {1,1,1,1} right]left[ {1,1,2} right]left[ {1,2,1} right]left[ {2,1,1} right]left[ {2,2} right] cr
    & left[ {1,1,1,1} right]left[ {1,1,2} right]left[ {1,2,1} right]left[ {2,1,1} right]left[ {2,2} right]left[ {1,3} right]left[ {3,1} right] cr
    & left[ {1,1,1,1} right]left[ {1,1,2} right]left[ {1,2,1} right]left[ {2,1,1} right]left[ {2,2} right]left[ {1,3} right]left[ {3,1} right]left[ 4 right] cr}
    $$



    Finally note that:

    - $N_{c,t}(n,k)$ correctly checks with OEIS seq. A126198, which provides further properties of these numbers;

    - the o.g.f. of $N_{c,t}$ in $n$ is in fact the $F(x)$ given by @jmerry's answer (re. to the o.g.f. for $N_b$ given in related post);
    $$
    sumlimits_{0, le ,,n} {N_{c,t} (n,k),x^{,n} } = {{1 - x} over {1 - 2x + x^{,k + 1} }}
    $$

    - $N_{c,t}(n,k)$ satisfies the recursion given by @NobleMushtak
    $$
    N_{c,t} (n,k) = sumlimits_{j = 1}^k {N_{c,t} (n - j,k)} + left[ {n = 0} right]
    $$

    where $[P]$ denotes the Iverson bracket.






    share|cite|improve this answer











    $endgroup$


















      1












      $begingroup$

      First a note about terminology:

      - a Partition of a positive integer $n$ is a non-decreasing sequence of positive integers summing to $n$;

      - a Composition of a positive integer $n$ is an unordered sequence of positive integers summing to $n$.



      That premised, you are speaking of the number of Compositions of $n$, whose terms (parts) are not-greater than $k$.



      Consider the case in which you seek for the number of compositions of the positive number $s+m$
      into $m$ parts not exceeding $r+1$
      $$
      {rm No}{rm .},{rm of},{rm solutions},{rm to};left{ matrix{
      {rm 1} le {rm integer};y_{,j} le r + 1 hfill cr
      y_{,1} + y_{,2} + ; cdots ; + y_{,m} = s + m hfill cr} right.
      $$

      that's the same as
      $$N_{,b} (s,r,m) = text{No}text{. of solutions to};left{ begin{gathered}
      0 leqslant text{integer }x_{,j} leqslant r hfill \
      x_{,1} + x_{,2} + cdots + x_{,m} = s hfill \
      end{gathered} right.$$



      $N_b$ is given by the following sum
      $$
      N_b (s,r,m)quad left| {;0 leqslant text{integers }s,m,r} right.quad =
      sumlimits_{left( {0, leqslant } right),,j,,left( { leqslant ,frac{s}{r+1}, leqslant ,m} right)}
      {left( { - 1} right)^k binom{m}{j}
      binom
      { s + m - 1 - jleft( {r + 1} right) }
      { s - jleft( {r + 1} right)} }
      $$

      as widely explained in this related post.



      Going back to your case, the
      number of compositions of $n$ into $m$ parts not greater than $k$

      will then be
      $$
      eqalign{
      & N_c (n,k,m)quad left| {;1 le n,m,k} right.quad = cr
      & = sumlimits_{left( {0, le } right),,j,,left( {, le ,m} right)} {left( { - 1} right)^{,j} binom{m}{j}
      binom{n - 1 - j,k}{ n - m - j,k}} cr}
      $$

      while the
      overall number of compositions of $n$ into parts not greater than $k$

      will be the sum
      of the above for $0 le m$ : if the sum extends over $n$ it will maintain the value $2^{n-1}$.
      $$ bbox[lightyellow] {
      eqalign{
      & N_{c,t} (n,k)quad left| {;0 le n,k in mathbb Z} right.quad = cr
      & = sumlimits_{0, le ,,m} {;sumlimits_{left( {0, le } right),,j,,left( {, le ,m} right)} {left( { - 1} right)^{,j}
      binom{m}{j} binom{ n - 1 - j,k}{n - m - j,k}
      } } cr}
      }$$



      In the example with $n=4$, the above gives for $k=1,2,3,4$
      $$
      1,5,7,8
      $$

      which in fact correspond to
      $$
      eqalign{
      & left[ {1,1,1,1} right] cr
      & left[ {1,1,1,1} right]left[ {1,1,2} right]left[ {1,2,1} right]left[ {2,1,1} right]left[ {2,2} right] cr
      & left[ {1,1,1,1} right]left[ {1,1,2} right]left[ {1,2,1} right]left[ {2,1,1} right]left[ {2,2} right]left[ {1,3} right]left[ {3,1} right] cr
      & left[ {1,1,1,1} right]left[ {1,1,2} right]left[ {1,2,1} right]left[ {2,1,1} right]left[ {2,2} right]left[ {1,3} right]left[ {3,1} right]left[ 4 right] cr}
      $$



      Finally note that:

      - $N_{c,t}(n,k)$ correctly checks with OEIS seq. A126198, which provides further properties of these numbers;

      - the o.g.f. of $N_{c,t}$ in $n$ is in fact the $F(x)$ given by @jmerry's answer (re. to the o.g.f. for $N_b$ given in related post);
      $$
      sumlimits_{0, le ,,n} {N_{c,t} (n,k),x^{,n} } = {{1 - x} over {1 - 2x + x^{,k + 1} }}
      $$

      - $N_{c,t}(n,k)$ satisfies the recursion given by @NobleMushtak
      $$
      N_{c,t} (n,k) = sumlimits_{j = 1}^k {N_{c,t} (n - j,k)} + left[ {n = 0} right]
      $$

      where $[P]$ denotes the Iverson bracket.






      share|cite|improve this answer











      $endgroup$
















        1












        1








        1





        $begingroup$

        First a note about terminology:

        - a Partition of a positive integer $n$ is a non-decreasing sequence of positive integers summing to $n$;

        - a Composition of a positive integer $n$ is an unordered sequence of positive integers summing to $n$.



        That premised, you are speaking of the number of Compositions of $n$, whose terms (parts) are not-greater than $k$.



        Consider the case in which you seek for the number of compositions of the positive number $s+m$
        into $m$ parts not exceeding $r+1$
        $$
        {rm No}{rm .},{rm of},{rm solutions},{rm to};left{ matrix{
        {rm 1} le {rm integer};y_{,j} le r + 1 hfill cr
        y_{,1} + y_{,2} + ; cdots ; + y_{,m} = s + m hfill cr} right.
        $$

        that's the same as
        $$N_{,b} (s,r,m) = text{No}text{. of solutions to};left{ begin{gathered}
        0 leqslant text{integer }x_{,j} leqslant r hfill \
        x_{,1} + x_{,2} + cdots + x_{,m} = s hfill \
        end{gathered} right.$$



        $N_b$ is given by the following sum
        $$
        N_b (s,r,m)quad left| {;0 leqslant text{integers }s,m,r} right.quad =
        sumlimits_{left( {0, leqslant } right),,j,,left( { leqslant ,frac{s}{r+1}, leqslant ,m} right)}
        {left( { - 1} right)^k binom{m}{j}
        binom
        { s + m - 1 - jleft( {r + 1} right) }
        { s - jleft( {r + 1} right)} }
        $$

        as widely explained in this related post.



        Going back to your case, the
        number of compositions of $n$ into $m$ parts not greater than $k$

        will then be
        $$
        eqalign{
        & N_c (n,k,m)quad left| {;1 le n,m,k} right.quad = cr
        & = sumlimits_{left( {0, le } right),,j,,left( {, le ,m} right)} {left( { - 1} right)^{,j} binom{m}{j}
        binom{n - 1 - j,k}{ n - m - j,k}} cr}
        $$

        while the
        overall number of compositions of $n$ into parts not greater than $k$

        will be the sum
        of the above for $0 le m$ : if the sum extends over $n$ it will maintain the value $2^{n-1}$.
        $$ bbox[lightyellow] {
        eqalign{
        & N_{c,t} (n,k)quad left| {;0 le n,k in mathbb Z} right.quad = cr
        & = sumlimits_{0, le ,,m} {;sumlimits_{left( {0, le } right),,j,,left( {, le ,m} right)} {left( { - 1} right)^{,j}
        binom{m}{j} binom{ n - 1 - j,k}{n - m - j,k}
        } } cr}
        }$$



        In the example with $n=4$, the above gives for $k=1,2,3,4$
        $$
        1,5,7,8
        $$

        which in fact correspond to
        $$
        eqalign{
        & left[ {1,1,1,1} right] cr
        & left[ {1,1,1,1} right]left[ {1,1,2} right]left[ {1,2,1} right]left[ {2,1,1} right]left[ {2,2} right] cr
        & left[ {1,1,1,1} right]left[ {1,1,2} right]left[ {1,2,1} right]left[ {2,1,1} right]left[ {2,2} right]left[ {1,3} right]left[ {3,1} right] cr
        & left[ {1,1,1,1} right]left[ {1,1,2} right]left[ {1,2,1} right]left[ {2,1,1} right]left[ {2,2} right]left[ {1,3} right]left[ {3,1} right]left[ 4 right] cr}
        $$



        Finally note that:

        - $N_{c,t}(n,k)$ correctly checks with OEIS seq. A126198, which provides further properties of these numbers;

        - the o.g.f. of $N_{c,t}$ in $n$ is in fact the $F(x)$ given by @jmerry's answer (re. to the o.g.f. for $N_b$ given in related post);
        $$
        sumlimits_{0, le ,,n} {N_{c,t} (n,k),x^{,n} } = {{1 - x} over {1 - 2x + x^{,k + 1} }}
        $$

        - $N_{c,t}(n,k)$ satisfies the recursion given by @NobleMushtak
        $$
        N_{c,t} (n,k) = sumlimits_{j = 1}^k {N_{c,t} (n - j,k)} + left[ {n = 0} right]
        $$

        where $[P]$ denotes the Iverson bracket.






        share|cite|improve this answer











        $endgroup$



        First a note about terminology:

        - a Partition of a positive integer $n$ is a non-decreasing sequence of positive integers summing to $n$;

        - a Composition of a positive integer $n$ is an unordered sequence of positive integers summing to $n$.



        That premised, you are speaking of the number of Compositions of $n$, whose terms (parts) are not-greater than $k$.



        Consider the case in which you seek for the number of compositions of the positive number $s+m$
        into $m$ parts not exceeding $r+1$
        $$
        {rm No}{rm .},{rm of},{rm solutions},{rm to};left{ matrix{
        {rm 1} le {rm integer};y_{,j} le r + 1 hfill cr
        y_{,1} + y_{,2} + ; cdots ; + y_{,m} = s + m hfill cr} right.
        $$

        that's the same as
        $$N_{,b} (s,r,m) = text{No}text{. of solutions to};left{ begin{gathered}
        0 leqslant text{integer }x_{,j} leqslant r hfill \
        x_{,1} + x_{,2} + cdots + x_{,m} = s hfill \
        end{gathered} right.$$



        $N_b$ is given by the following sum
        $$
        N_b (s,r,m)quad left| {;0 leqslant text{integers }s,m,r} right.quad =
        sumlimits_{left( {0, leqslant } right),,j,,left( { leqslant ,frac{s}{r+1}, leqslant ,m} right)}
        {left( { - 1} right)^k binom{m}{j}
        binom
        { s + m - 1 - jleft( {r + 1} right) }
        { s - jleft( {r + 1} right)} }
        $$

        as widely explained in this related post.



        Going back to your case, the
        number of compositions of $n$ into $m$ parts not greater than $k$

        will then be
        $$
        eqalign{
        & N_c (n,k,m)quad left| {;1 le n,m,k} right.quad = cr
        & = sumlimits_{left( {0, le } right),,j,,left( {, le ,m} right)} {left( { - 1} right)^{,j} binom{m}{j}
        binom{n - 1 - j,k}{ n - m - j,k}} cr}
        $$

        while the
        overall number of compositions of $n$ into parts not greater than $k$

        will be the sum
        of the above for $0 le m$ : if the sum extends over $n$ it will maintain the value $2^{n-1}$.
        $$ bbox[lightyellow] {
        eqalign{
        & N_{c,t} (n,k)quad left| {;0 le n,k in mathbb Z} right.quad = cr
        & = sumlimits_{0, le ,,m} {;sumlimits_{left( {0, le } right),,j,,left( {, le ,m} right)} {left( { - 1} right)^{,j}
        binom{m}{j} binom{ n - 1 - j,k}{n - m - j,k}
        } } cr}
        }$$



        In the example with $n=4$, the above gives for $k=1,2,3,4$
        $$
        1,5,7,8
        $$

        which in fact correspond to
        $$
        eqalign{
        & left[ {1,1,1,1} right] cr
        & left[ {1,1,1,1} right]left[ {1,1,2} right]left[ {1,2,1} right]left[ {2,1,1} right]left[ {2,2} right] cr
        & left[ {1,1,1,1} right]left[ {1,1,2} right]left[ {1,2,1} right]left[ {2,1,1} right]left[ {2,2} right]left[ {1,3} right]left[ {3,1} right] cr
        & left[ {1,1,1,1} right]left[ {1,1,2} right]left[ {1,2,1} right]left[ {2,1,1} right]left[ {2,2} right]left[ {1,3} right]left[ {3,1} right]left[ 4 right] cr}
        $$



        Finally note that:

        - $N_{c,t}(n,k)$ correctly checks with OEIS seq. A126198, which provides further properties of these numbers;

        - the o.g.f. of $N_{c,t}$ in $n$ is in fact the $F(x)$ given by @jmerry's answer (re. to the o.g.f. for $N_b$ given in related post);
        $$
        sumlimits_{0, le ,,n} {N_{c,t} (n,k),x^{,n} } = {{1 - x} over {1 - 2x + x^{,k + 1} }}
        $$

        - $N_{c,t}(n,k)$ satisfies the recursion given by @NobleMushtak
        $$
        N_{c,t} (n,k) = sumlimits_{j = 1}^k {N_{c,t} (n - j,k)} + left[ {n = 0} right]
        $$

        where $[P]$ denotes the Iverson bracket.







        share|cite|improve this answer














        share|cite|improve this answer



        share|cite|improve this answer








        edited Dec 30 '18 at 18:57

























        answered Dec 29 '18 at 22:35









        G CabG Cab

        20.5k31342




        20.5k31342






























            draft saved

            draft discarded




















































            Thanks for contributing an answer to Mathematics Stack Exchange!


            • Please be sure to answer the question. Provide details and share your research!

            But avoid



            • Asking for help, clarification, or responding to other answers.

            • Making statements based on opinion; back them up with references or personal experience.


            Use MathJax to format equations. MathJax reference.


            To learn more, see our tips on writing great answers.




            draft saved


            draft discarded














            StackExchange.ready(
            function () {
            StackExchange.openid.initPostLogin('.new-post-login', 'https%3a%2f%2fmath.stackexchange.com%2fquestions%2f3055314%2fnumber-of-compositions-of-n-such-that-each-term-is-less-than-equal-to-k%23new-answer', 'question_page');
            }
            );

            Post as a guest















            Required, but never shown





















































            Required, but never shown














            Required, but never shown












            Required, but never shown







            Required, but never shown

































            Required, but never shown














            Required, but never shown












            Required, but never shown







            Required, but never shown







            Popular posts from this blog

            Biblatex bibliography style without URLs when DOI exists (in Overleaf with Zotero bibliography)

            ComboBox Display Member on multiple fields

            Is it possible to collect Nectar points via Trainline?